LSAT and Law School Admissions Forum

Get expert LSAT preparation and law school admissions advice from PowerScore Test Preparation.

 Administrator
PowerScore Staff
  • PowerScore Staff
  • Posts: 8916
  • Joined: Feb 02, 2011
|
#24973
Complete Question Explanation

Flaw in the Reasoning. The correct answer choice is (A)

In this stimulus, the author asserts that the view that every person is only concerned with his or her own self interest implies that government by consent is impossible. The author does not support this connection in any way whatsoever. He or she simply states it as a fact. The argument continues from this unsupported statement to state that social theorists who think that people are only concerned with their own self-interest must also think that government by consent is impossible. At no point in the argument does the author provide support for the claim that the first belief requires the second. The correct answer choice will address this lack of support.

This question is a wonderful example of how the language in the answer choices can make a question much more difficult than it would otherwise seem. As explained above, the main flaw is related to a generalized lack of support for the conclusion. Identifying that flaw as a prephrase is only half the battle; we will also need to parse terse language of the answer choices to find the one that describes the correct flaw.

Answer choice (A): This is the correct answer choice. It states that the conclusion is unsupported by pointing out that even if one has a certain view, they may not believe all the implications of that view. Looking back at the stimulus, we can see that the author fails to support the idea that social theorists who believe that people are only concerned with their own self interest are also required to believe that government by consent is impossible. Therefore, since this answer choice correctly describes the flaw in the stimulus, it is correct.

Answer choice (B): This is one of two answer choices that incorrectly identify this stimulus as having an error in composition/division. This answer choice states the mistake was to claim that because a group had a certain quality or belief, each member of the group needed to hold that belief. In this stimulus, the author does not draw broad generalizations about large groups of people and then attempt to apply those generalizations to every single individual member of the group. Rather, he or she identifies a group as social theorists all of whom hold a particular view. Therefore, this is not the flaw.

Answer choice (C): Like in answer choice (B), this answer choice references errors of composition/division. The conclusion of the argument is not a broad generalization about a group, and thus, the answer choice is incorrect.

Answer choice (D): This answer choice describes an ad hoc attack on the proponents of a viewpoint. The author of the stimulus does not attack others personal qualities, for example by stating that the proponents of a view are wrong because they are short sighted, or anti-American. The author does not personalize the attack at all. Therefore, this answer choice is incorrect.

Answer choice (E): At no point in the stimulus does the author reference assumptions. This answer choice does not describe the argument in the stimulus, so it has to be incorrect.
 jared.xu
  • Posts: 65
  • Joined: Oct 07, 2011
|
#3165
This is a flaw question. But I couldn't recognize the flaw, and so I choose C, thinking it might be composition error. I understand why the answer is A now. But what "common errors of reasoning" would answer A fit under? Thank you.
User avatar
 Dave Killoran
PowerScore Staff
  • PowerScore Staff
  • Posts: 5852
  • Joined: Mar 25, 2011
|
#3166
Hi Jared,

Thanks for the question. Remember, the common errors of reasoning are just the ones that appear most frequently, and because they are used so much, being able to identify them is extremely helpful. But, they are just the "common" ones, not every single error in existence (indeed, such a list would be impossible to make, and would be so long that it would be largely useless).

Answer choice (A) in this problem is probably closest to a type of unwarranted assumption. It is the type of answer (in its details, at least) that you probably won't ever see on the LSAT again.

Please let me know if that helps. Thanks!
 sinclab
  • Posts: 1
  • Joined: Oct 16, 2014
|
#17082
Hi, I am having difficulty identifying the specific flaw in the reasoning in this problem. I chose answer choice C, but the correct answer is A. The wording of A isn't identifiable in the 17 different Flaw options listed in the LR Bible so I think that's why I'm having difficulty. Would someone help me understand why A is correct? Thanks!!
 Lucas Moreau
PowerScore Staff
  • PowerScore Staff
  • Posts: 216
  • Joined: Dec 13, 2012
|
#17084
Hello, sinclab,

Yes, I can see the difficulty with this question. This is a non-standard Flaw question. :0 But the flaw in question is in fact one you've learned before.

This is an error of overgeneralization flaw. It is incorrectly assuming that since a person holds one belief, that person must hold all implications of that belief also. It's worded oddly, but that's the flaw.

Here's an analogy: If a person is British, this implies that this person loves to drink tea over coffee. Thus, British people would never choose to go to a coffee shop over a tea shop.

Being British, here, implies that a person loves to drink tea rather than coffee, but it is not "must be true" true that every British person loves drinking tea rather than coffee.

In the question, just because holding the view that every person is concerned exclusively with their own self interest implies that government by consent is impossible, the fact that someone holds that view is not necessarily proof that that person also holds the view that government by consent is impossible. Many people might hold the view that everyone only cares about their own self interest, but that it is in their self interest to have a government that operates by consent that a democracy - maybe enough to sustain a whole democracy. ;)

Hope that helps,
Lucas Moreau
 akalsi
  • Posts: 34
  • Joined: Aug 25, 2014
|
#30828
Hi,

Could you provide an example where answer choice E would apply? I'm understanding the situations in which the other answer choices would come up, but I can't seem to wrap my head around a situation where answer choice E would be the correct one. Also, is this a common type of flaw that we learned?

Thanks in advance!
 David Boyle
PowerScore Staff
  • PowerScore Staff
  • Posts: 836
  • Joined: Jun 07, 2013
|
#30860
akalsi wrote:Hi,

Could you provide an example where answer choice E would apply? I'm understanding the situations in which the other answer choices would come up, but I can't seem to wrap my head around a situation where answer choice E would be the correct one. Also, is this a common type of flaw that we learned?

Thanks in advance!

Hello akalsi,

Re answer E, "fails to consider that, even if an argument’s conclusion is false, some of the assumptions used to justify that conclusion may nonetheless be true": I don't think it's that common a type of flaw, although you could call it a species of "error of division". I.e., just because an argument as a whole is wrong, that doesn't automatically mean that every element of the argument is wrong. Or, you could also call it a sort of overgeneralization (some of the argument is wrong, so the rest of it must be wrong). Or, it could even be deemed a sort of weird reverse version of one of the "errors in the use of evidence". E.g., if you took the error "lack of evidence for a position is taken to prove that position is false", and strangely reversed it to say, "a false position is taken to mean that all evidence for that position is lacking in truth", that might resemble answer E.

What kind of argument might even go there, though? How could you make an argument that followed what answer E says? Well, say, if you take "assumption" as being able to mean a stated premise (so that we can even list it and talk about it below), instead of being an unstated premise:
Joe Blow saw a sign that said, "Donald Trump has orange hair. Therefore, the world is going to end." Joe Blow then said, "Well, that's ridiculous. We can't prove the world is going to end just because of the Donald's hair color. Therefore, even the assumption that he has orange hair is incorrect." However, I, Jane Jones, note that Joe Blow failed to consider that even if that argument on the sign has a conclusion that's false, some of the assumptions used to justify that argument's conclusion may nonetheless be true. E.g., Trump may actually have orange hair."
Something like that might work.

Hope this helps,
David
 egarcia193
  • Posts: 41
  • Joined: Jun 25, 2017
|
#41191
Hi
When we see both types of composition answer choices, such as B and C in the answers would that automatically eliminate both of them being correct. Since they are both listed in the answers?
 Claire Horan
PowerScore Staff
  • PowerScore Staff
  • Posts: 408
  • Joined: Apr 18, 2016
|
#42114
Hi Egarcia193,

Kudos to you for comparing answer choices to see if the presence of both allows you to eliminate each one. For this particular situation, though, that won't work because they describe different scenarios. Errors of composition and errors of division are distinct types of errors.

Scenario #1:
Premise: Dalmatians are the least healthy species of dog.
Conclusion: That dalmation must be less healthy than that chihuahua.

Answer choice (B) describes this scenario: (B) infers that because something is true of a group of people, it is true of each individual member of the group.

Scenario #2:
Premise: Every atom in a penny is invisible to the naked eye.
Conclusion: The penny itself must also not be visible to the naked eye.

Answer choice (C) describes this scenario: (C) infers that because something is true of each individual person belonging to a group, it is true of the group as a whole.

Thanks for the question!
 Cobblestone
  • Posts: 3
  • Joined: Jan 13, 2022
|
#93359
Hi,

I am having difficulty piecing together the various explanations provided for this question. The first explanation, as I read it, seems to read that because this is not a broad generalisation flaw, answer choices B and C are incorrect. Yet there is another explanation (October 2014), which states that this is an "error of over-generalisation."
I do not believe that the stimulus contained a generalisation, so the first explanation resonates more with me, yet I am not sure how it relates to the October 2014 explanation.
Any thoughts on this would be much appreciated. Thanks!

Get the most out of your LSAT Prep Plus subscription.

Analyze and track your performance with our Testing and Analytics Package.